Podłoga równanie

Permutacje. Kombinacje. Wariacje. Rozmieszczanie kul w urnach. Silnie i symbole Newtona. Przeliczanie zbiorów. Funkcje tworzące. Teoria grafów.
ibefree
Użytkownik
Użytkownik
Posty: 43
Rejestracja: 5 wrz 2011, o 15:07
Płeć: Mężczyzna
Lokalizacja: Polska
Podziękował: 4 razy

Podłoga równanie

Post autor: ibefree »

Musze udowodnic ze prawdziwe jest wyrazenie.

\(\displaystyle{ \left\lfloor \frac{\left\lfloor{x}\right\rfloor}{k} \right\rfloor = \left\lfloor \frac{x}{k} \right\rfloor}\)

\(\displaystyle{ x,k \in Z}\) - calkowite
Ostatnio zmieniony 7 wrz 2011, o 15:41 przez ibefree, łącznie zmieniany 2 razy.
kipsztal
Użytkownik
Użytkownik
Posty: 54
Rejestracja: 4 wrz 2011, o 17:55
Płeć: Mężczyzna
Lokalizacja: ALL WORLD
Podziękował: 2 razy
Pomógł: 2 razy

Podłoga równanie

Post autor: kipsztal »

tak na oko x i k dowolne
ibefree
Użytkownik
Użytkownik
Posty: 43
Rejestracja: 5 wrz 2011, o 15:07
Płeć: Mężczyzna
Lokalizacja: Polska
Podziękował: 4 razy

Podłoga równanie

Post autor: ibefree »

ktoś może to rozpisac ?
Awatar użytkownika
yorgin
Użytkownik
Użytkownik
Posty: 12762
Rejestracja: 14 paź 2006, o 12:09
Płeć: Mężczyzna
Lokalizacja: Kraków
Podziękował: 17 razy
Pomógł: 3440 razy

Podłoga równanie

Post autor: yorgin »

Jeżeli \(\displaystyle{ x\in\mathbb{Z}}\) to nie ma czego dowodzić. Sprawdź dokładnie treść.
Użytkownik
Użytkownik
Posty: 9833
Rejestracja: 18 gru 2007, o 03:54
Płeć: Mężczyzna
Lokalizacja: Bydgoszcz
Podziękował: 90 razy
Pomógł: 2632 razy

Podłoga równanie

Post autor: »

\(\displaystyle{ x}\) oczywiście może być dowolne.

Oznaczmy:
\(\displaystyle{ \left\lfloor \frac{\left\lfloor{x}\right\rfloor}{k} \right\rfloor =n \in \mathbb{Z}}\)
Wówczas z definicji mamy:
\(\displaystyle{ n\le \frac{\left\lfloor{x}\right\rfloor}{k}< n+1\\
kn\le \left\lfloor{x}\right\rfloor < kn+k}\)

To jednak jest równoważne (z własności 3.7 w Matematyce konkretnej lub z chwili zastanowienia ;) ) :
\(\displaystyle{ kn\le x<kn+k}\)
czyli
\(\displaystyle{ n\le\frac xk<n+1}\)
a to z definicji znaczy, że:
\(\displaystyle{ \left\lfloor \frac{x}{k} \right\rfloor = n}\)
co kończy dowód równości.

Q.
ODPOWIEDZ